• Shuffle
    Toggle On
    Toggle Off
  • Alphabetize
    Toggle On
    Toggle Off
  • Front First
    Toggle On
    Toggle Off
  • Both Sides
    Toggle On
    Toggle Off
  • Read
    Toggle On
    Toggle Off
Reading...
Front

Card Range To Study

through

image

Play button

image

Play button

image

Progress

1/296

Click to flip

Use LEFT and RIGHT arrow keys to navigate between flashcards;

Use UP and DOWN arrow keys to flip the card;

H to show hint;

A reads text to speech;

296 Cards in this Set

  • Front
  • Back
In which vessel would longitudinal smooth muscle be most abundant?
a. a renal arteriole
b. a skin capillary
c. a leg vein
d. the aorta
e. a cerebral artery
c. a leg vein
In which vessel would elastic fibers be most abundant?
a. a renal arteriole
b. a skin capillary
c. a leg vein
d. the aorta
e. a cerebral artery
d. the aorta
Which of the following would you expect to occur if the diameter of an arteriole in your big toe decreases by 10% ?
a. Your toe would turn green
b. Your heart rate would increase
c. Your toe would have increased perfusion
d. Your skin would get red
e. None of the above
e. None of the above
Which of the following would be most likely to result from aortic stenosis ?
a. chronic hypertension
b. sinus bradycardia
c. diastolic murmurs
d. S3
d. S3
Which of the following would NOT reduce ventricular filling?
a. restrictive cardiomyopathy
b. mitral stenosis
c. constriction of venous vascular smooth muscle
d. pericardial tamponade
e. a dramatic increase in heart rate to 180 beats per minute
c. constriction of venous vascular smooth muscle
Exercise-induced tachycardia occurs with which of the following diseases?:
a. AV block
b. digitalis toxicity
c. mutations in Ryanodine receptor (Ryr)
d. vasovagal syndrome
c. mutations in Ryanodine receptor (Ryr)
A defect in conduction through the AV node producing a third degree AV block would be
would be expected to cause:
a. pulse rate of 100 bprn with SA node firing of 80 bprn
b. pulse rate and SA node firing rates of 100 bprn
c. pulse rate of 60 bprn with SA node firing rate of 100 bprn
d. pulse rate and SA node firing rate of 60 bprn
c. pulse rate of 60 bprn with SA node firing rate of 100 bprn
Re-entry in ventricular muscle would cause:
a. sinus tachycardia
b. sinus bradycardia
c. tachycardia with ectopic beats
d. no change in heart rate
c. tachycardia with ectopic beats
Opening of L type Calcium channels is directly caused by:
a. binding of specific ligands such as acetylcholine to the channel
b. an increase in voltage across the plasma membrane (depolarization)
c. activation by ryanodine
d. activation by cGMP
e. none of the above
b. an increase in voltage across the plasma membrane (depolarization)
An increase in cardiac contractility can occur as a result of:
a. dilated cardiomyopathy
b. an increase in blood volume
c. treatment with L-type calcium channel blockers
d. sympathetic stimulation
e. all of the above
d. sympathetic stimulation
Hypertension may contribute to the development of:
a. aortic valve insufficiency
b. atherosclerosis
c. cardiac hypertrophy
d. myocardial ischemia
e. all of the above
e. all of the above
Which of the following would be most consistent with poor wound healing, leg pain and leg ulcers in a 50 year old male smoker?
a. Buerger's disease
b. Raynaud's syndrome
c. thrombophlebitis
d. coarctation of the aorta
e. patent ductus arteriosus
a. Buerger's disease
Which of the following is consistent with aortic murmur, high systolic pressure measured in the brachial artery, and pain in the leg with exercise?
a. Buerger's disease
b. Raynaud's syndrome
c. thrombophlebitis
d. coarctation of the aorta
e. patent ductus arteriosus
d. coarctation of the aorta
Which of the following would be found in a fatty streak?
a. lipid deposits within the endothelial cells
b. lipid deposits in the smooth muscle cells in the muscular layer of the vessel wall
c. lipid deposits in the macrophages in the intimal layer
d. lipid deposits in platelets in the intimal layer
c. lipid deposits in the macrophages in the intimal layer
Which of the following could be used to block progression of atherosclerosis?:
a. a drug which inhibits macrophage synthesis of cytokines
b. a drug which stimulates scavenger receptor expression
c. a drug which stimulates oxidation of LDLs
d. a drug which inhibits HDL synthesis
a. a drug which inhibits macrophage synthesis of cytokines
Which of the following is true concerning the mechanism of action of a Beta-1 agonist drug on the heart:
a. Beta-1 agonists increase dephosphorylation of phospholamban in cardiac myocytes
b. Beta-1 agonists increase phosphorylation of ryanodine receptors in cardiac myocytes
c. Beta-l agonists increase phosphorylation of Big K channels in cardiac myocytes
d. Beta-l agonists decrease the rate of Na+ entry into SA node cells
b. Beta-1 agonists increase phosphorylation of ryanodine receptors in cardiac myocytes
Which of the following statements about control of blood flow is true:
a. The cerebral circulation autoregulates its flow over a range of pressures
b. Skeletal muscle blood flow during exercise is controlled exclusively by an increase in sympathetic stimulation.
c. There is no autoregulation in the kidney vasculature.
d. Acetycholine causes vasoconstriction in sweat glands.
a. The cerebral circulation autoregulates its flow over a range of pressures
Phenylephrine is a drug which is an agonist at alpha 1 receptors. Which of the following would you expect to occur after phenylephrine is injected? (Be sure to consider reflexes)
a. a decrease in blood pressure and heart rate
b. an increase in blood pressure and heart rate
c. an increase in blood pressure and decrease in heart rate
d. a decrease in blood pressure and increase in heart rate
e. an increase in blood pressure but no change in heart rate
c. an increase in blood pressure and decrease in heart rate
Which is true about sympathetic stimulation in the cardiovascular system?
a. in normal people it is the major factor maintaining heart rate at rest
b. in normal people it is the major factor maintaining vascular smooth muscle tone
c. in patients with myocardial ischemia, sympathetic stimulation of the SA node is reduced
d. in patients with low blood volume due to hemorrhage, sympathetic stimulation of the venous vasculature is reduced
b. in normal people it is the major factor maintaining vascular smooth muscle tone
Which of the following would you expect to be true in a normal patient after loss of 700 ml of blood?
a. Blood flow in the brain would be decreased.
b. Blood flow to the skin would be decreased.
c. Blood flow in the coronary blood vessels would be decreased.
d. Stimulation of the SA node by sympathetic nerves would be decreased
b. Blood flow to the skin would be decreased.
In a patient with severe hypertropic cardiomyopathy, stroke volume would:
a. increase
b. decrease
c. not change
b. decrease
Myocardial ischemia could be caused in a patient with coronary artery disease when stimulation of the heart:
a. increase
b. decrease
c. not change
a. increase
Atrial natriuretic peptide causes the tone (degree of constriction) of the vascular smooth muscle cell to:
a. increase
b. decrease
c. not change
b. decrease
In a patient with coarctation of the aorta mean arterial pressure in the femoral artery would: (relative to a normal person)
a. increase
b. decrease
c. not change
b. decrease
Cardiac hypertrophy causes the O2 demand of the heart to:
a. increase
b. decrease
c. not change
a. increase
In response to an occlusion in a coronary artery, autoregulation would cause the relaxation of vascular smooth muscle in downstream arterioles to:
a. increase
b. decrease
c. not change
a. increase
Ventricular fibrillation would cause the blood pressure to:
a. increase
b. decrease
c. not change
b. decrease
Increased sympathetic firing to the heart would cause permeability of calcium channels in the SA node to:
a. increase
b. decrease
c. not change
a. increase
An increase in K+ efflux out of a cardiac muscle cell causes the membrane potential of that
cell to:
a. increase
b. decrease
c. not change
b. decrease
Vasoconstrictors cause intracellular Ca++ levels in vascular smooth muscle cells to:
a. increase
b. decrease
c. not change
a. increase
Viagra decreases the breakdown of cGMP in vascular smooth muscle; this would cause the blood flow in the vessels with vasodilator stimulation to:
a. increase
b. decrease
c. not change
a. increase
Absolute refractory period ends near the start of this phase:
a. phase 0
b. phase 1
c. phase 2
d. phase 3
e. phase 4
d. phase 3
The timing of this phase is delayed in the form of long QT syndrome caused by Kdr channel mutations
a. phase 0
b. phase 1
c. phase 2
d. phase 3
e. phase 4
d. phase 3
Digitalis will cause after-depolarizations during this phase:
a. phase 0
b. phase 1
c. phase 2
d. phase 3
e. phase 4
e. phase 4
L-type Ca++ channels are open during this phase
a. phase 0
b. phase 1
c. phase 2
d. phase 3
e. phase 4
c. phase 2
Kto channels open during this phase
a. phase 0
b. phase 1
c. phase 2
d. phase 3
e. phase 4
b. phase 1
Na+ activation gates open at the start and close at the end of this phase
a. phase 0
b. phase 1
c. phase 2
d. phase 3
e. phase 4
a. phase 0
When would the following occur?
Left ventricular volume is greatest
a. end of diastole/start of systole
b. end of systole/start of diastole
a. end of diastole/start of systole
When would the following occur?
Greatest decrease in left ventricular pressure begins
a. end of diastole/start of systole
b. end of systole/start of diastole
b. end of systole/start of diastole
When would the following occur?
The mitral valve closes
a. end of diastole/start of systole
b. end of systole/start of diastole
a. end of diastole/start of systole
When would the following occur?
The aortic valve closes
a. end of diastole/start of systole
b. end of systole/start of diastole
b. end of systole/start of diastole
Indicate the effect on vascular
smooth muscle:
Nitroprusside (NO donor)
a. constriction
b. dilation
b. dilation
Indicate the effect on vascular
smooth muscle:
PGE2
a. constriction
b. dilation
b. dilation
Indicate the effect on vascular
smooth muscle:
Angiotensin II
a. constriction
b. dilation
a. constriction
Indicate the effect on vascular smooth muscle:
Thromboxane
a. constriction
b. dilation
a. constriction
Indicate the effect on vascular
smooth muscle:
Adenosine
a. constriction
b. dilation
b. dilation
An increase in parasympathetic nerve firing to the heart would cause cardiac output to:
a. increase
b. decrease
c. no change
b. decrease
An increase in firing in the sympathetic nerves to the veins would cause cardiac output to:
a. increase
b. decrease
c. no change
a. increase
In a patient with restrictive cardiomyopathy, filling of the heart would:
a. increase
b. decrease
c. no change
b. decrease
Atrial natriuretic peptide causes the tone (degree of constriction) of the vascular smooth muscle cell to:
a. increase
b. decrease
c. no change
b. decrease
In a patient with postductal coarctation of the aorta, mean arterial pressure in the femoral artery would : (relative to a normal person)
a. increase
b. decrease
c. no change
b. decrease
Viagra decreases the breakdown of cGMP in vascular smooth muscle; this would cause the blood flow in the vessels with vasodilator stimulation to:
a. increase
b. decrease
c. no change
a. increase
In response to an occlusion in a coronary artery, the tone (constriction) of the downstream arterioles will:
a. increase
b. decrease
c. no change
a. increase
Cardiac hypertrophy causes the O2 demand of the heart to:
a. increase
b. decrease
c. no change
a. increase
During exercise, the consumption of O2 causes blood flow to the exercising muscle to:
a. increase
b. decrease
c. no change
a. increase
Platelets release substances which generally cause tone of the vessel to:
a. increase
b. decrease
c. no change
a. increase
An increase in K+outside of a cardiac muscle cell will cause the membrane potential of the cell to:
a. increase
b. decrease
c. no change
a. increase
An increase in K+ efflux out of a cardiac muscle cell will cause the membrane potential of that cell to:
a. increase
b. decrease
c. no change
b. decrease
Ca++ channels are open
a. phase 0
b. phase 1
c. phase 2
d. phase 3
e. phase 4
c. phase 2
Absolute refractory period ends near the start of this phase
a. phase 0
b. phase 1
c. phase 2
d. phase 3
e. phase 4
d. phase 3
Digitalis will cause after-depolarizations during this phase
a. phase 0
b. phase 1
c. phase 2
d. phase 3
e. phase 4
e. phase 4
Inward flux of Na+
a. phase 0
b. phase 1
c. phase 2
d. phase 3
e. phase 4
a. phase 0
The period of the cardiac cycle during which the aortic valve is closed and the mitral valve is opened.
a. isovolumetric contraction
b. systolic ejection
c. isovolumetric relaxation
d. diastolic filling
d. diastolic filling
The period of the cardiac cycle during which aortic pressure increases the most.
a. isovolumetric contraction
b. systolic ejection
c. isovolumetric relaxation
d. diastolic filling
b. systolic ejection
The second heart sound occurs at the beginning of this part of the cycle.
a. isovolumetric contraction
b. systolic ejection
c. isovolumetric relaxation
d. diastolic filling
c. isovolumetric relaxation
EDV in the ventricle would occur immediately before the start of this part of the cycle.
a. isovolumetric contraction
b. systolic ejection
c. isovolumetric relaxation
d. diastolic filling
a. isovolumetric contraction
Elastic fibers are abundant in which of the following vessels?
a. the aorta
b. capillaries
c. arterioles
d. small venules
a. the aorta
Which of the following is TRUE regarding valve defects?
a. Mitral valve stenosis results in a systolic murmur.
b. Mitral valve regurgitation results in a systolic murmur.
c. Aortic valve stenosis results in a diastolic murmur.
d. Aortic valve regurgitation results in a systolic murmur.
b. Mitral valve regurgitation results in a systolic murmur.
Aortic stenosis would be likely to result in:
a. left atrial dilation
b. left atrial hypertrophy
c. left ventricular dilation
d. left ventricular hypertrophy
d. left ventricular hypertrophy
An increase in the interval from the P wave to the QRS is likely to be caused by:
a. an increase in parasympathetic stimulation of the SA node
b. an increase in parasympathetic stimulation of the AV node
c. an increase in sympathetic firing to ventricular muscle cells
d. an increase in sympathetic firing to the SA node
b. an increase in parasympathetic stimulation of the AV node
Re-entry in ventricular muscle would cause:
a. sinus tachycardia
b. sinus bradycardia
c. tachycardia with ectopic beats
d. no change in heart rate
c. tachycardia with ectopic beats
The pacemaker potential in the heart:
a. requires sympathetic stimulation to occur
b. occurs because of conductance (flow) of Na+ ions into the cells
c. is the result of the very negative resting membrane potential of the cells
d. is increased by parasympathetic stimulation
b. occurs because of conductance (flow) of Na+ ions into the cells
Which of the following is NOT required in order to produce a coordinated pumping by the ventricles?
a. an intact bundle of His and Purkinje system
b. gap junctions between ventricular muscle fibers
c. a normally acting AV node
d. all of the above are required
c. a normally acting AV node
OR
d. all of the above are required
Which of the following is NOT TRUE of the mechanism of action of a Beta-1 drug on the heart:
a. It works by increasing phosphorylation of phospholamban in myocytes
b. It works by increasing phosphorylation of RyR in myocytes
c. It works by increasing phosphorylation of Big K channels in myocytes
d. It works by increasing the rate of Na+ entry into SA node cells
c. It works by increasing phosphorylation of Big K channels in myocytes
Which of the following is likely to increase EDV?
a. infusion of saline intravenously
b. infusion of an alpha-1 agonist
c. riding a bicycle
d. all of the above
d. all of the above
The ryanodine receptor (RyR) linked channel is NOT important for which of the following:
a. vascular smooth muscle cell relaxation
b. voltage-dependent Ca++ influx
c. Ca++ -stimulated Ca++ release
d. IP3 stimulated Ca++ release
b. voltage-dependent Ca++ influx
Which of the following changes in the ECG would occur in a patient with a defect in the delayed rectifier K+ channel?
a. longer time between P and QRS
b. wide QRS
c. QT interval increased
d. irregularly shaped P waves
c. QT interval increased
Which of the following changes in the ECG would occur in a patient with a right bundle branch defect?
a. longer time between P and QRS
b. wide QRS
c. ST segment elevated
d. irregularly shaped P waves
b. wide QRS
Which of the following would NOT reduce both ventricular filling and stroke volume?
a. atrial fibrillation
b. mitral stenosis
c. decreased venous compliance
d. pericardial effusion
c. decreased venous compliance
An increase in stroke volume above the normal level would be caused by which of the following:
a. infusion of a Beta-1 antagonist (beta-blocker)
b. an increase in filling of the heart EDV
c. an increase in afterload
d. all of the above
b. an increase in filling of the heart EDV
Which of the following would be found in a fatty streak?
a. lipid deposits within the endothelial cells
b. lipid deposits in the smooth muscle cells in the tunica media
c. lipid deposits in the macrophages in the tunica intima
d. lipid deposits in platelets in the tunica intima
c. lipid deposits in the macrophages in the tunica intima
Which symptoms are consistent with Raynaud’s phenomenon?
a. edema and hyperpigmentation in the lower limb
b. a systolic murmur and elevated heart rate
c. vasospasms with exposure to cold
d. thin shiny skin and thick nails; risk of gangrene
c. vasospasms with exposure to cold
Which of the following is NOT true about aneurysms?
a. The risk is increased by hypertension.
b. They can be related to congenital or genetic defects.
c. The formation is related to wall tension.
d. They are likely only in small vessels.
d. They are likely only in small vessels.
Indomethacin or ibuprofen are used to treat infants with patent ductus arteriosus because:
a. They stimulate blood clotting to cause the ductus to close.
b. They cause pulmonary vasodilation to reduce flow form the aorta to the pulmonary artery.
c. They inhibit PGE2 production, resulting in ductal vasoconstriction.
d. They inhibit PGF2alpha which causes ductal dilation.
c. They inhibit PGE2 production, resulting in ductal vasoconstriction.
Which of the following disease states or defects is least likely to cause pulmonary hypertension?
a. mitral stenosis
b. aortic regurgitation
c. dilated cardiomyopathy
d. mitral valve prolapse
d. mitral valve prolapse
Which sensor mechanism would be most important for the increase in heart rate you feel if you go to the top of Pike’s Peak (elevation 14,000 ft)?
a. aortic barorecepors
b. atrial B receptors
c. carotid chemoreceptors
d. ventricular stretch receptors
c. carotid chemoreceptors
Angiotensin has been implicated in the process of hypertrophy in the heart. Use of converting enzyme inhibitors (which reduce production of angiotensin) would be most useful in treating which of the following patients?
a. patients after myocardial infarction
b. hypertensive patients
c. patients with chronic severe aortic regurgitation
d. all of the above
d. all of the above
Which of the following would occur following clot formation in several vessels supplying a tissue?
a. Total peripheral resistance would decrease, and the tissue would appear blue.
b. Total peripheral resistance would not change because heart rate would increase.
c. Total peripheral resistance would increase, and the tissue would be pale.
d. Total peripheral resistance would not change, although the tissue would become necrotic.
b. Total peripheral resistance would not change because heart rate would increase.
AN is a 65 year old woman who presents in the hospital with cool skin, sweaty and in an agitated state. Her pulse is 90 beats per minute and irregular; her blood pressure is 110/64 mmHg. On further examination a pansystolic murmur was detected. An ECG shows small irregular P waves. A more detailed history indicates that she has a long history of “heart murmur” and exercise intolerance. She says she has been feeling more tired lately and has frequent palpitations, especially after she walks or does yard work.
Which of the following diagnoses would be consistent with the presentation in AN?
a. She has restrictive cardiomyopathy.
b. She has mitral regurgitation.
c. She has systemic hypotension.
d. None of the above are consistent with her signs and symptoms.
b. She has mitral regurgitation.
AN is a 65 year old woman who presents in the hospital with cool skin, sweaty and in an agitated state. Her pulse is 90 beats per minute and irregular; her blood pressure is 110/64 mmHg. On further examination a pansystolic murmur was detected. An ECG shows small irregular P waves. A more detailed history indicates that she has a long history of “heart murmur” and exercise intolerance. She says she has been feeling more tired lately and has frequent palpitations, especially after she walks or does yard work.
Her ECG pattern is consistent with which of the following?
a. sinus arrhythmia
b. ventricular tachycardia
c. atrial flutter
d. a primary AV nodal block
c. atrial flutter
AN is a 65 year old woman who presents in the hospital with cool skin, sweaty and in an agitated state. Her pulse is 90 beats per minute and irregular; her blood pressure is 110/64 mmHg. On further examination a pansystolic murmur was detected. An ECG shows small irregular P waves. A more detailed history indicates that she has a long history of “heart murmur” and exercise intolerance. She says she has been feeling more tired lately and has frequent palpitations, especially after she walks or does yard work.
Which of the following would most likely be the underlying etiology of her disease?
a. rheumatic heart disease
b. hypertension
c. atherosclerosis
d. pericarditis
a. rheumatic heart disease
AN is a 65 year old woman who presents in the hospital with cool skin, sweaty and in an agitated state. Her pulse is 90 beats per minute and irregular; her blood pressure is 110/64 mmHg. On further examination a pansystolic murmur was detected. An ECG shows small irregular P waves. A more detailed history indicates that she has a long history of “heart murmur” and exercise intolerance. She says she has been feeling more tired lately and has frequent palpitations, especially after she walks or does yard work.
Which of the following are possible long term complications if she is not adequately treated?
a. pulmonary edema
b. embolization
c. myocardial ischemia
d. all of the above
d. all of the above
Which of the following is true regarding valve defects?
a. Mitral valve stenosis results in a systolic murmur
b. Mitral valve regurgitation results in a diastolic murmur
c. Aortic valve stenosis results in a systolic murmur
d. Aortic valve regurgitation results in a systolic murmur
c. Aortic valve stenosis results in a systolic murmur
Which of the following is NOT TRUE about regulation of blood flow to tissues?
a. Blood flow to the skin can be decreased by increased sympathetic activity
b. Blood flow to the brain is indirectly altered by changes in resistance in other tissues and directly altered by autoregulation
c. Blood flow to the kidney is affected by sympathetic nerves and autoregulation
d. Blood flow in the coronary arteries is always independent of blood pressure
e. Blood flow to the salivary glands can be increased by parasympathetic stimulation
d. Blood flow in the coronary arteries is always independent of blood pressure
Which of the following would NOT cause contractility to decrease?
a. a loss of pi receptors
b. dilated Cardiomyopathy
c. a decrease in venous return
d. a person with myocardial ischemia
c. a decrease in venous return
An increase in the interval from the p wave to the QRS is likely to be related to a change in:
a. An increase in parasympathetic stimulation of the SA node
b. An increase in parasympathetic stimulation of the AV node
c. An increase in sympathetic firing to ventricular muscle cells
d. An increase in sympathetic firing to the SA node
b. An increase in parasympathetic stimulation of the AV node
A change in the shape of the T wave in the ECG is most likely related to a change in extracellular concentrations of which ion?
a. Na
b. Ca
c. Cr
d. K
d. K
Reentry can be caused by which of the following
a. formation of a fibrotic scar
b. an abnormality in the sodium channel resulting in prolongation of the refractory period
c. ischemia in myocytes
d. both b and c
d. both b and c
Spread of contraction over the atria depends on:
a. innervation by the sympathetic nervous system of atrial myocytes
b. gap junctions between atrial myocytes
c. presence of t-tublues between myocytes
d. the bundle of His which is a high speed conduction system between the myocytes
b. gap junctions between atrial myocytes
The resting (or pacemaker) potential in the SA node cells is caused by
a. an inward leak of Na+
b. an outward leak of Na
c. Opening of K channels
d. Opening of L-type Ca channels
a. an inward leak of Na+
Delayed rectifier K+ channels are important for which phase(s) of the action potential in ventricular myocytes
a. phase 4 only
b. phase 1 only
c. phase 0 to 1
d. phase 2-3
d. phase 2-3
Which of the following channels is NOT voltage-dependent?
a. The Na+ channel on ventricular muscle cells
b. The Na+ channel on SA node cells
c. The ryanodine coupled Ca++ channel in ventricular muscle cells
d. The L-Type Ca++ channel on ventricular muscle cells
c. The ryanodine coupled Ca++ channel in ventricular muscle cells
Which of the following would you expect as a consequence of early after depolarization?
a. atrial fibrillation
b. AV block
c. Torsades de pointes
d. Sinus tachycardia
c. Torsades de pointes
Mitral stenosis would be likely to cause
a. atrial dilation
b. a murmur between SI and S2
c. increased left ventricular end diastolic pressure
d. increased cardiac output
a. atrial dilation
Administration of antibiotics in patients with mitral valve prolapse is intended to
a. reduce the risk of pericarditis
b. reduce the risk of blood clots
c. reduce the risk of endocarditis
d. prevent angina
c. reduce the risk of endocarditis
Vascular smooth muscle vasoconstriction would be increased by an agent which
a. increases IP3 stimulation of ryanodine receptors
b. decreases phosphorylation of L-type Ca++ channel
c. increases phosphorylation of Ca++ ATPase
d. increases opening of "big" K+channels
a. increases IP3 stimulation
Indomethacin or ibuprofen are used to treat infants with patent ductus arteriosus because
a. They stimulate blood clotting to cause the ductus to close
b. They cause pulmonary vasodilation to reduce flow from the aorta to the pulmonary artery
c. They inhibit PGF2-alpha which causes ductal dilation and decreases blood pressure in the infants
d. They inhibit PGE2 production, resulting in ductal vasoconstriction.
c. They inhibit PGF2-alpha which causes ductal dilation and decreases blood pressure in the infants
In a patient the infusion of an al agonist resulted in a small increase in cardiac output. Which of the following is most likely to explain this effect on cardiac output?
a. Heart rate was increased by an activation of al receptors
b. Contractility was increased by an activation of al receptors
c. Venous return was increased by an activation of al receptors
d. There was an increase in heart rate as a reflex response to the increase in blood pressure which occurred because of activation of al receptors
c. Venous return was increased by an activation of al receptors
Which of the following are thought to be involved in the appearance of fatty streaks in atherosclerosis?
a. presence of platelets
b. release of cytokines by smooth muscle cells
c. uptake of oxidized LDLs
d. presence of adhesion molecules on the macrophages
c. uptake of oxidized LDLs
A venous thrombosus would cause all of the following except
a. tenderness and swelling
b. pain at the site of thrombosus
c. pale skin
d. neuralgia
c. pale skin
Agents which inhibit activity of the Na+/K+ ATPase can increase contractility if given at the proper (therapeutic doses). This increase in contractility would occur because
a. Intracellular K+ levels increase
b. Intracelluar Na+ levels decrease
c. Intracellular Ca++ levels increase
d. Ventricular volume increases
c. Intracellular Ca++ levels increase
Which of the following is most important for the relaxation of the ventricular myocardium
a. K to channels
b. Ca ATPase on plasma membrane
c. Ca ATPase on sarcoplasmic reticulum
d. L-type Ca++ channel
c. Ca ATPase on sarcoplasmic reticulum
Which of the following are NOT actions of the sympathetic nervous system on ventricular myocytes:
a. increase activity of protein kinases
b. increasing resting membrane potential
c. increase phosphorylation of L-type Ca channels
d. increase phosphorylation of phospholamban
b. increasing resting membrane potential
Afterload may contribute to the development of
a. aortic valve insufficiency
b. cardiac hypertrophy
c. myocardial ischemia
d. all of the above
d. all of the above
Which of the following would NOT be associated with Raynaud's disease:
a. intermittent flow to fingertips and/or toes
b. thrombosis
c. thick, brittle nails
d. numbness
b. thrombosis
Which of the following diseases tend to occur in patient with other autoimmune diseases
a. Arteritis
b. Buerger's disease
c. coarctation of the aorta
d. aortic stenosis
a. Arteritis
In which of these tissues is autoregulation of blood flow most important
a. The skin
b. The stomach
c. The brain
d. The liver
c. The brain
Restrictive cardiomypathy limits cardiac function because
a. contractility is reduced
b. mitral valve regurgitation occurs
c. diastolic filling is reduced
d. heart rate is decreased
c. diastolic filling is reduced
Which of the following alter contractility
a. Norepinephrine
b. pi agonists
c. agents that increase cAMP generation in ventricular muscle
d. all of the above
d. all of the above
A patient is infused with a drug; as a result, the following changes are measured in the
cardiovascular system:
heart rate to increased from 75 to 80 beats per minute cardiac output increased from 4.5 to 4.8 liters
arterial blood pressure decreased from 90 mmHg to 85 mmHg.
Which of the following occurred in this patient? (assume no change in right atrial pressure)
a. Neither stroke volume nor resistance changed
b. Stroke volume and resistance both increased
c. Resistance decreased with no change in stroke volume
d. Resistance decreased with an increased in stroke volume
c. Resistance decreased with no change in stroke volume
A patient is infused with a drug; as a result, the following changes are measured in the
cardiovascular system:
heart rate to increased from 75 to 80 beats per minute cardiac output increased from 4.5 to 4.8 liters
arterial blood pressure decreased from 90 mmHg to 85 mmHg.
In this patient, the drug was most likely
a. a vasodilator like nitroprusside
b. a vasoconstrictor like angiotensin II
c. a cholinergic receptor agonist
d. a beta receptor antagonist
a. a vasodilator like nitroprusside
A 17 year old male complains to the doctor that he has had a few syncopal episodes when he is exercising. The patient otherwise feels healthy. The doctor does a thorough physical and finds that he has a systolic murmur. He is referred to a pediatric cardiologist who confirms the presence of systolic murmur. He also notes a fourth heart sound. He performs an echocardiogram and an ECG. The results of these tests lead the doctor to diagnose the patient with hypertrophic cardiomyopathy.
The cause of hypertropic cardiomyopathy in this patient is likely to be
a. Smoking
b. poor diet during childhood
c. genetic defect
d. Atherosclerosis
c. genetic defect
A 17 year old male complains to the doctor that he has had a few syncopal episodes when he is exercising. The patient otherwise feels healthy. The doctor does a thorough physical and finds that he has a systolic murmur. He is referred to a pediatric cardiologist who confirms the presence of systolic murmur. He also notes a fourth heart sound. He performs an echocardiogram and an ECG. The results of these tests lead the doctor to diagnose the patient with hypertrophic cardiomyopathy.
Which of the following would be expected to be seen with echocardiography
a. an enlarged, dilated ventricle
b. calcium deposits in the ventricular wall
c. dramatically increased ventricular wall thickness
d. a thickened pericardium
c. dramatically increased ventricular wall thickness
A 17 year old male complains to the doctor that he has had a few syncopal episodes when he is exercising. The patient otherwise feels healthy. The doctor does a thorough physical and finds that he has a systolic murmur. He is referred to a pediatric cardiologist who confirms the presence of systolic murmur. He also notes a fourth heart sound. He performs an echocardiogram and an ECG. The results of these tests lead the doctor to diagnose the patient with hypertrophic cardiomyopathy.
The most likely cause of the systolic murmur is
a. mitral regurgitation
b. aortic regurgitation
c. mitral stenosis
d. patent ductus arteriosus
a. mitral regurgitation
A 17 year old male complains to the doctor that he has had a few syncopal episodes when he is exercising. The patient otherwise feels healthy. The doctor does a thorough physical and finds that he has a systolic murmur. He is referred to a pediatric cardiologist who confirms the presence of systolic murmur. He also notes a fourth heart sound. He performs an echocardiogram and an ECG. The results of these tests lead the doctor to diagnose the patient with hypertrophic cardiomyopathy.
54. Patients with this disease can develop atrial fibrillation. This is likely to be related to
a. increased parasympathetic tone to AV node
b. sinus tachycardia
c. increased plasma K+
d. increased atrial volume and pressure
d. increased atrial volume and pressure
A 17 year old male complains to the doctor that he has had a few syncopal episodes when he is exercising. The patient otherwise feels healthy. The doctor does a thorough physical and finds that he has a systolic murmur. He is referred to a pediatric cardiologist who confirms the presence of systolic murmur. He also notes a fourth heart sound. He performs an echocardiogram and an ECG. The results of these tests lead the doctor to diagnose the patient with hypertrophic cardiomyopathy.
Patients with this disease are frequently treated with beta blockers (beta antagonists). This would be useful because:
a. beta blockers reduce risk of myocardial ischemia
b. beta blockers would reduce the risk of tachycardia developing with exercise
c. beta blockers would help increase the time for filling of the heart.
d. all of the above
d. all of the above
A 50 year old male appears in the emergency room complaining of chest pain that began about 90 minutes ago. He also is somewhat short of breath and is feeling sick to his stomach, although he has not vomited. A history is taken: he has taken his blood pressure at the local pharmacy over the past few months and says that it averages about 140/90. His blood cholesterol levels have been elevated (about 240). He jogs 2 miles 3 times a week and says he has been getting little short of breath lately when he does so.
A physical exam is performed. The patient has a blood pressure of 160/98. His pulse rate is 98 beats per minute. He is also tachypneic. No murmurs or extra heart sounds are noted. An ECG is performed. The diagnosis is myocardial ischemia.
The most likely underlying cause for the ischemia in this patient is:
a. cardiac hypertrophy
b. Atherosclerosis
c. hypertension
d. b and c
e. all of the above
d. b and c
A 50 year old male appears in the emergency room complaining of chest pain that began about 90 minutes ago. He also is somewhat short of breath and is feeling sick to his stomach, although he has not vomited. A history is taken: he has taken his blood pressure at the local pharmacy over the past few months and says that it averages about 140/90. His blood cholesterol levels have been elevated (about 240). He jogs 2 miles 3 times a week and says he has been getting little short of breath lately when he does so.
A physical exam is performed. The patient has a blood pressure of 160/98. His pulse rate is 98 beats per minute. He is also tachypneic. No murmurs or extra heart sounds are noted. An ECG is performed. The diagnosis is myocardial ischemia.
Which of the following would you expect to find on ECG, given the patient's present condition
a. change in the T wave such as ST segment elevation
b. atrial fibrillation
c. ventricular fibrillation
d. AV block
a. change in the T wave such as ST segment elevation
A 50 year old male appears in the emergency room complaining of chest pain that began about 90 minutes ago. He also is somewhat short of breath and is feeling sick to his stomach, although he has not vomited. A history is taken: he has taken his blood pressure at the local pharmacy over the past few months and says that it averages about 140/90. His blood cholesterol levels have been elevated (about 240). He jogs 2 miles 3 times a week and says he has been getting little short of breath lately when he does so.
A physical exam is performed. The patient has a blood pressure of 160/98. His pulse rate is 98 beats per minute. He is also tachypneic. No murmurs or extra heart sounds are noted. An ECG is performed. The diagnosis is myocardial ischemia.
The ejection fraction in the patient is measured to be 30% (normal about 60%). What explains the increased blood pressure?
a. high venous tone
b. high contractility
c. high arteriolar resistance
d. high heart rate
e. all of the above
c. high arteriolar resistance
A 50 year old male appears in the emergency room complaining of chest pain that began about 90 minutes ago. He also is somewhat short of breath and is feeling sick to his stomach, although he has not vomited. A history is taken: he has taken his blood pressure at the local pharmacy over the past few months and says that it averages about 140/90. His blood cholesterol levels have been elevated (about 240). He jogs 2 miles 3 times a week and says he has been getting little short of breath lately when he does so.
A physical exam is performed. The patient has a blood pressure of 160/98. His pulse rate is 98 beats per minute. He is also tachypneic. No murmurs or extra heart sounds are noted. An ECG is performed. The diagnosis is myocardial ischemia.
The patient is given nitrates. These will improve the patient's condition because
a. heart rate will decrease
b. myocardial blood flow will increase
c. blood flow to peripheral tissues will increase
d. a and b
e. a and c
b. myocardial blood flow will increase
A 50 year old male appears in the emergency room complaining of chest pain that began about 90 minutes ago. He also is somewhat short of breath and is feeling sick to his stomach, although he has not vomited. A history is taken: he has taken his blood pressure at the local pharmacy over the past few months and says that it averages about 140/90. His blood cholesterol levels have been elevated (about 240). He jogs 2 miles 3 times a week and says he has been getting little short of breath lately when he does so.
A physical exam is performed. The patient has a blood pressure of 160/98. His pulse rate is 98 beats per minute. He is also tachypneic. No murmurs or extra heart sounds are noted. An ECG is performed. The diagnosis is myocardial ischemia.
Aspirin is also used to treat this patient because
a. It would be effective at limiting pain due to angina
b. It would reduce LDL oxidation
c. It would reduce clot formation
d. It only has a placebo effect, but it will reduce anxiety in the patient
c. It would reduce clot formation
Aortic valve closure causes which heart sound?________________
S2 or second
Atrial fibrillation is most likely to result from stenosis of the _________ valve.
mitral
Opening of L type Calcium channels directly cause:
a. Calcium entry into the ventricular cell across the plasma membrane
b. Calcium release from SR
c. both of the above
a. Calcium entry into the ventricular cell across the plasma membrane
Increased sympathetic firing to the heart would cause heart rate to (increase, decrease, not change)_______________ because permeability of calcium channels in the SA node (increase, decrease, not change)_______________.
increase
increase
If the radius of an arteriole in the kidney decreases, then resistance will (increase, decrease, not change)_______________, and flow to the kidney will (increase, decrease, not change)_______________.
increase
decrease
Mitral valve closure causes which heart sound?_____________
S1 or first
Mitral stenosis results in a murmur during _____________
diastole
Ventricular hypertrophy is most likely to result from stenosis of the _________ valve.
aortic
Opening of L type Calcium channels is directly caused by:
a. binding of specific ligands like acetylcholine to the channel
b. Change in voltage across the plasma membrane
c. none of the above
b. Change in voltage across the plasma membrane
Increased parasympathetic firing to the heart would cause heart rate to (increase, decrease, not change)________________ because permeability of the leaky sodium channels in the SA node (increase, decrease, not change) _________________
decrease
decrease
If the radius of a arteriole in the kidney increases, then resistance will (increase, decrease, not change)_______________, and flow to the kidney will (increase, decrease, not change)_______________.
decrease
increase
Pressure in arterioles is (more/less) _______________ than pressure in the veins.
more
During diastole, pressure in the ventricle is (more/less) _______________ than pressure in the aorta.
less
The absolute refractory period in ventricular muscle cells is:
a. shorter than that in nerves
b. determined by the permeability of the L type Calcium channels
c. determined by timing of inactivation gates on the sodium channels
d. determined by potassium channel opening
c. determined by timing of inactivation gates on the sodium channels
The first heart sound is caused by
a. closure of the AV valves
b. opening of the AV valves
c. closure of the aortic and pulmonic valves
d. opening of the aortic and pulmonic valves
a. closure of the AV valves
If the radius of a arteriole to the small intestine increases, then resistance will (increase, decrease, not change)___________________, and flow to the small intestine will (increase, decrease, not change)___________________.
decrease
increase
Compliance in veins is (more/less) _______________ than compliance in arterioles.
more
End diastolic volume in the venticles (more/less) _______________ than end systolic volume.
more
The pacemaker potential is:
a. a more negative potential than in most nerves
b. determined by the permeability of the L_type Calcium channels
c. dependent on leakiness of cationic channels which allow sodium entry
d. dependent on potassium channel opening
c. dependent on leakiness of cationic channels which allow sodium entry
The second heart sound is caused by
a. closure of the AV valves
b. opening of the AV valves
c. closure of the aortic and pulmonic valves
d. opening of the aortic and pulmonic valves
c. closure of the aortic and pulmonic valves
If the radius of an arteriole to the spleen decreases, then resistance will (increase, decrease, not change)_____________________, and flow to the spleen will (increase, decrease, not change)_____________________.
increase
decrease
If resistance in an arteriole goes up, then the flow to the tissue supplied by this arteriole must (increase or decrease) __________________.
decrease
The type of blood vessel with high compliance is a _______________.
vein
Closure of the mitral valve causes the _______ heart sound. (S1, S2, S3 or S4?)
S1 or first
A stenotic aortic valve will cause a murmur during __________________ (systole or diastole).
systole
Heart rate is decreased by increased _____________ activity (parasympathetic or sympathetic).
parasympathetic
In the ventricular muscle cell: The Phase 0 of action potential is caused primarily by influx of ________________ (K+, Na+, or Ca++).
Phase 2 is caused by influx of ___________ (K+, Na+, or Ca++) channels through ______________ (type of channels).
Na+
Ca++
L-type calcium channels
The SERCA pumps is located where? ________ (Cell and location in cell). It pumps what ion? ___________.
ventricular muscle cell or vascular smooth muscle cell; SR
Ca++
An increase in firing in the parasympathetic nerve to the heart would cause heart rate to ______________(increase/decrease)
decrease
An increase in contractility of the ventricles is can be caused by ____________
The concentration of which ion in the cell is responsible for changes in contractility ___________.
NE, epi, Beta 1, inotropic drugs
Ca++
Dilated cardiomyopathy causes contractility to _____________ (increase/decrease), which would generally cause cardiac output and blood pressure to _____________ (increase/decrease)
decrease
decrease
Ventricular fibrillation would cause the blood pressure to ______________ (increase/decrease).
decrease
Vasodilation can be caused by _____________ (name one agent), which causes resting membrane potential to ______________ (increase=less negative/decrease=more negative).
any dilator: NE. ANP, PGE2, NO, prostacyclin
decrease
A drug which acts on Beta-1 receptors in the heart would ______________ (increase/decrease) heart rate.
increase
Dilation of the atrium would increase the risk of:
a. sinus tachycardia
b. early afterdepolarizations
c. atrial fibrillation
d. a widened QRS complex
c. atrial fibrillation
Restrictive cardiomyopathy results in decreased _________________ (filling/strength of contraction)
filling
The Frank Starling mechanism says that stroke volume will increase with increases in preload. Preload increases with an increase in _______________.
venous return
Vasoconstriction can be caused by _____________ (name one agent), which causes intracellular Ca++ levels to ______________ (increase/decrease).
any constrictor: NE, epi, vasopressin, angiotensin II, PGF2alpha, thromboxane, endothelin
increase
Regurgitation of the mitral valve causes a murmur during ______________ (systole or diastole).
systole
A stenotic aortic valve will cause a murmur during ______________(systole or diastole).
systole
Heart rate is decreased when the activity of the ___________ (parasympathetic or sympathetic) nerves to the heart increases.
parasympathetic
The SERCA pumps is located where in the heart? (Cell and location in cell) _________.
Increased activity of the SERCA causes ventricular muscle cells to ___________(contract/relax) more efficiently.
ventricular muscle cell and sarcoplamsmic reticulum
relax
An increase in contractility of the ventricles is can be caused by ______________.
The concentration of which ion in the cardiomyocyte is involved in the mechanism of contractility ______________.
sympathetic or epinephrine
Dilated cardiomyopathy causes contractility to ____________ (increase/decrease), which would generally cause cardiac output and blood pressure to ____________ (increase/decrease)
decrease
decrease
Re-entry results in:
a. sinus bradycardia
b. sinus tachycardia
c. atrial or ventricular tachycardia
d. long QT
c. atrial or ventricular tachycardia
If the radius of an arteriole increases then the resistance in this vessel will ____________ (increase/decrease/not change) and the flow in the vessel will _____________ (increase/decrease/not change).
decrease
increase
The vessel with the thinnest vessel wall would be the (a)____________ (aorta, renal artery, capillary in the liver, vena cava), whereas the vessel with the greatest compliance would be the (b)________ (renal artery, capillary in the liver, vena cava, carotid artery).
capillary in the liver
vena cava
In a patient with mitral stenosis the pressure in the left atrium would be _______________ (increased/decreased/no change) as compared to a person with a normal mitral valve.
increased
Volume in the heart at the end of diastole is _______________ (greater than, less than) the volume at the end of systole.
greater than
In a patient with mitral regurgitation the murmur would occur during __________ (systole, diastole).
systole
During isovolumetric contraction the pressure in the left ventricle (a) _______________ (increases/decreases/does not change), and at the peak of the contraction the pressure in the left ventricle is (b)_______________ (greater than, less than) the pressure in the aorta.
increases
greater than
The fourth heart sound would occur:
a. during ventricular relaxation
b. during atrial contraction
c. during rapid filling
d. during ventricular contraction
b. during atrial contraction
. An increase in sympathetic nerve firing to the heart would cause cardiac output to:
a. increase
b. decrease
c. no change
a. increase
. An increase in sympathetic nerve firing to arterioles in the gastointestinal tract would cause flow to the GI tract to:
a. increase
b. decrease
c. no change
b. decrease
An increase in venous compliance would cause stroke volume to:
a. increase
b. decrease
c. no change
b. decrease
In a patient with dilated cardiomypathy contractility would:
a. increase
b. decrease
c. no change
b. decrease
Blocking phospholamban action causes the TONE (degree of constriction) of the vascular smooth muscle cell to:
a. increase
b. decrease
c. no change
b. decrease
Angiotensin causes the TONE (degree of constriction) of the vascular smooth muscle cell to:
a. increase
b. decrease
c. no change
b. decrease
An increase in __________ions in the ventricular muscle fiber would cause contractility to increase.
Ca++
Hypertrophy can occur as an adaptation in the heart if volume work increases because of: (Name THREE possibilities) ____________________
mitral or aortic regurgitation or heart failure
Beta 2 receptors cause vasodilation in arterioles in what tissue? __________________
skeletal muscle
Which type of cardiomyopathy is usually genetic? __________________
hypertrophic
An increase in parasympathetic nerve firing to the heart would cause cardiac output to:
a. increase
b. decrease
c. no change
b. decrease
An increase in sympathetic nerve firing to arterioles in the kidney would cause flow to the kidney to:
a. increase
b. decrease
c. no change
b. decrease
In a patient with restricted cardiomypathy End Diastolic Volume would:
a. increase
b. decrease
c. no change
b. decrease
An infusion of saline intravenously would cause stroke volume to:
a. increase
b. decrease
c. no change
a. increase
A drug which acts by increasing IP3 generation in vascular smooth muscle would cause TONE (degree of constriction) of the vascular smooth muscle cell to:
a. increase
b. decrease
c. no change
a. increase
Atrial natriuretic peptide causes the TONE (degree of constriction) of the vascular smooth muscle cell to:
a. increase
b. decrease
c. no change
b. decrease
Beta 2 receptors would cause contractility to:
a. increase
b. decrease
c. no change
b. decrease
In a patient with a patent ductus arteriosus, a murmur is heard (when): _____________________
72 hours after birth
Which type of cardiomyopathy can occur in alcoholics? __________________
dilated cardiomyopathy
The first stage of atherosclerosis is called the ________________________ and involves migration of ____________( CELL TYPE) into the vascular wall.
Fatty streak
Monocytes (macrophages, lymphocytes or T cells)
If a vessel in the lower leg is occluded and the skin distal to the occlusion appears blue then the occlusion is in what type of vessel? _________________
Vein
In myocardial ischemia, changes occur in which portion of the ECG? _______. This portion of the ECG corresponds to what phase of the ventricular action potential? ______
T wave or ST segment
Phase 3 or K efflux or repolarization
Baroreceptors are located in the wall of the __________ and the __________ arteries.
carotid
aortic (or aortic arch)
If a patient is given a vasodilator, then you would expect systemic vascular resistance to _____________ (increase/decrease); the heart rate would ______________(increase/decrease) as a reflex response to the change in blood pressure.
decrease
increase
The second stage of atherosclerosis is called the ________________________ and involves migration of ____________(CELL TYPE) from lower layers into the intimal layer.
fibrous plaque
smooth muscle cells
If a vessel in the lower leg is occluded and the skin distal to the occlusion appears white or pale then the occlusion is in what type of vessel? _________________
artery or arteriole
In myocardial infaction, sympathetic activity is normally _____________(increased/decreased) and ventricular ____________ (tachycardia/bradycardia) is common.
increase
tachycardia
If a patient is given a vasoconstrictor, then you would expect systemic vascular resistance to _____________ (increase/decrease); the heart rate would ______________(increase/decrease) as a reflex response to the change in blood pressure.
Increase
Decrease
Which of the following is most likely to increase myocardial work
a) anemia
b) hemorrhage
c) increased arterial blood pressure
d) atherosclerosis
c) increased arterial blood pressure
In a newborn with a patent ductus, most commonly the flow in the ductus will be from (aorta to pulmonary artery, pulmonary artery to aorta).
aorta to pulmonary artery
Autoregulation would cause ____________ (vasodilation, vasocontriction) as a response to decreased pressure in the artery to that tissue. This effect may be mediated by ____________ (nitric oxide, angiotensin II, norepinephrine).
vasodilation
nitric oxide
Treatment with a drug which stimulates α1 receptors would cause blood pressure to ____________ (increase/ decrease). This effect occurs because there is a/an _________ (increase/decrease) in activity of the Ryr receptor/channel in vascular smooth muscle cells.
increase
increase
A decrease in firing in the parasympathetic nerve to the heart would cause heart rate to (a) ________________ (increase/decrease). This effect would result from a/an (b) _____________ (increase/decrease) in Na+ entry in phase 4 of the action potential in the SA node.
increase
increase
An increase in epinephrine would cause contractility to (a) _______________(increase/decrease) because the SR release of (b) _______________ (what ion) is increased.
increase
Ca++
. Increases in skeletal muscle activity in your legs would cause EDV to (a) _____________ (increase/decrease). This results in a/an (b) ___________ (increase/decrease) in stroke volume. This effect is called the (c) ____________.
increase
increase
Frank-Starling mechanism, effect, or law OR heterotropic autoregualtion OR intrinsic control
Vasodilation can be caused by (a) _____________ (name one agent), which causes resting membrane potential to (b) ________________ (increase= less negative / decrease= more negative).
any dilator: NE. ANP, PGE2, NO, prostacyclin, adenosine
decrease
The valve which closes at the beginning of ventricular contraction is the _________ valve; this produces the ______ heart sound.
mitral
first
Volume in the heart at the end of systole is _______ (greater than/less than) the volume at the end of diasystole.
less than
In a patient with chronic aortic stenosis, the pressure in the left ventricle would be ________ (increased /decreased/not changed) as compared to a person with a normal aortic valve.
increased
In a patient with mitral regurgitation the murmur would occur during __________ (systole, diastole).
systole
The channel in the heart responsible for Phase 2 of the ventricular muscle action potential is the ________ channel (specific).
L type Ca++ channel
Torsades de pointes occurs as a result of depolarization at the _________ phase of the cardiac action potential, and causes ventricular heart rate to __________ (increase/decrease/not change).
end of phase 2 OR start of phase 3
increase
A beta1 agonist would cause the threshold for Ca++ channel opening in the SA node to _______ (increase /decrease/not change), and ventricular muscle SR Ca++ release to __________ (increase/decrease/not change).
decrease
increase
If sympathetic nerve firing to the kidney increases, then the flow to the kidney would be expected to (increase or decrease) _________, but aortic blood pressure would (increase or decrease) _________.
decrease
increase
During a hemorrhage, the cardiac output would (increase or decrease) _________ and heart rate would (reflexly) (increase or decrease) _________
decrease
increase
In atherosclerosis, in the fatty streak stage the lipid deposition occurs because of uptake of fat from (what type of carrier molecule) ___________ by (what type of cell-give name before uptake)________________
LDL (or oxidized LDL), monocyte (or macrophage)
When a coronary artery is occluded, the normal response in the vessel distal to the occlusion would be _________ (dilation/constriction); a factor/chemical that would be likely to cause this response is_______________.
dilation
adenosine (or K+, CO2, low O2)
If a myocardial infarct occurs, a short term response to help maintain flow would be an (increase or decrease) _________
in sympathetic nerve activity, whereas a very long term response would be an increase in myocardial ____________.
increase
hypertrophy (size, mass)
A drug is given which DIRECTLY decreases blood pressure but doesn't directly change cardiac output. This drug is most likely to be:
a. a beta agonist
b. an alpha agonist
c. nitroprusside
d. vasopressin
c. nitroprusside
Within seconds after administering the drug (same as above), heart rate increases and cardiac output increases. This reflex response occurs because:
a. baroreceptor firing increases and causes increased sympathetic stimulation of the SA node
b. baroreceptor firing increases and causes increased parasympathetic stimulation of the SA node
c. baroreceptor firing decreases and causes increased sympathetic stimulation of the SA node
d. baroreceptor firing decreases and causes increased parasympathetic stimulation of the SA node
c. baroreceptor firing decreases and causes increased sympathetic stimulation of the SA node
During a hemorrhage, the cardiac output would __________ (increase / decrease) and heart rate would (reflexly) ________ (increase / decrease).
decrease
increase
Injection of an alpha agonist would cause blood pressure to _________ (increase / decrease / not change), which would cause heart rate AND contractility (reflexly) to __________ (increase / decrease / not change).
increase
decrease
When the ductus remains patent postnatally, the most common direction of flow is _________ (left to right / right to left) because pressure in the pulmonary artery is usually ______ (less than / greater than ) pressure in the aorta.
Left to right
Less than
Aortic stenosis could result in __________ (dilated cardiomyopathy OR hypertropic cardiomyopathy OR concentric hypertrophy), because of __________ (poor alignment of muscle fibers OR increased left ventricular pressure).
concentric hypertrophy
increased left ventricular pressure
Dilated cardiomyopathy causes contractility to ______________ (increase / decrease), which would generally cause cardiac output and blood pressure to ______________ (increase / decrease).
decrease
decrease
Infusion of a nitroprusside would cause resistance to ______________ (increase/decrease) and heart rate to ______________(increase/decrease).
decrease
increase
The final stage of atherosclerosis is called the _____________ and involves migration and involvement of ____________(CELL TYPE) at the site of the plaque.
complicated lesion
platelets
In a newborn with a patent ductus, most commonly the flow in the ductus will be from __________ (aorta to pulmonary artery, pulmonary artery to aorta) because pressure is greatest in the __________ (aorta, pulmonary artery).
aorta to pulmonary artery
aorta
In postductal coarctaction of the aorta, brachial pressure is ________________(higher than, less than, the same as) pressure in the femoral artery, and plasma angiotensin would be _____________(increased, decreased, no change).
higher than
increased
Pulse in the ankle would be faint in __________ (disease name).
Buerger’s disease OR arteriosclerosis obliterans OR coarctation of the aorta
In myocardial ischemia, changes occur in which portion of the ECG? _______. This portion of the ECG corresponds to what phase of the ventricular action potential? ______
T wave OR ST segment OR ST
phase 3 OR K+ efflux
Myocardial work will increase if left ventricular pressure _______ (increases, decreases, no change) such as with the valve defect ________.
increases
aortic stenosis
In a normal heart, coronary flow increases with increased work. This is because of increased _________ production in the exercising cardiac muscle.
adenosine OR lactic acid
During diastole, pressure in the ventricle is (more/less) ___________ than pressure in the aorta.
less
End diastolic volume in the venticles (more/less) ___________ than end systolic volume.
more
If the radius of a arteriole to the small intestine increases, then resistance will (increase, decrease, not change) [a.] ___________, and flow to the small intestine will (increase, decrease, not change)
[b.] __________.
decrease
increase
If the aortic valve is stiff it causes [a.] __________, which will result in an [b.] ____________ (increase, decrease, not change) in ventricular pressure and a [c.] ___________ (increase, decrease) in stroke volume.
stenosis
Increase
decrease
The absolute refractory period in ventricular muscle cells is:
a. shorter than that in nerves
b. determined by the permeability of the L type Calcium channels
c. determined by timing of inactivation gates on the sodium channels
d. determined by potassium channel opening
c. determined by timing of inactivation gates on the sodium channels
The first heart sound is caused by:
a. closure of the AV valves
b. opening of the AV valves
c. closure of the aortic and pulmonic valves
d. opening of the aortic and pulmonic valves
a. closure of the AV valves
Vasoconstriction is increased when the activity of the _________ receptors in arterioles is stimulated by norepinephrine.
alpha 1
An increase in contractility of the ventricles can be caused by _______________.
The concentration of which ion in the cardiomyocyte is involved in the mechanism of contractility ____________.
sympathetic or epinephrine
Ca++
Dilated cardiomyopathy causes contractility to _______________ (increase/decrease), which would generally cause cardiac output and blood pressure to _____________ (increase/decrease)
decrease
decrease
If blood pressure decreases, the firing of the carotid sinus baroreceptor nerve will ___________________. The response to this is a/an ___________ in firing in the vagus (parasympathetic) nerve to the heart, resulting in a/an ___________ in heart rate.
decrease
decrease
increase
The first stage of atherosclerosis is called the ____________ and involves formation of ___________ (CELL TYPE) within the vascular wall.
fatty streak
foam cells
During exercise, increased flow to the cardiac muscle is mediated by the locally produced agent ___________ which acts to cause ____________ (vasodilation, vasocontriction) as a response to (increased/decreased) ___________ myocardial work.
adenosine
vasodilation
increase
The vascular disease in which blood vessels in the hands may alternatively dilate and constrict producing changes in color of the finger tips is ____________. This disease is can be related to _____________ (smoking, congenital defect, cold exposure).
Raynaud’s
cold exposure
A patient with 80% occlusion of his/her main coronary artery is most likely to have ___________ (stable angina, unstable angina), which occurs ________ (with exercise, at rest and with exercise).
stable
with exercise
D. is a 40 year old woman who normally has a blood pressure of 110/70 and heart rate of 65 bpm. She has a headache and takes a drug from her medicine cupboard which she thinks is aspirin, but she mistakenly takes another drug from the cupboard. She rests but within an hour she notices that her heart rate has increased. She takes her blood pressure and it has increased to 122/78.
Which of the following agents would be consistent with this response?
a. A beta antagonist
b. An alpha agonist
c. An alpha antagonist
d. A mixed alpha and beta agonist
e. An angiotensin agonist
d. A mixed alpha and beta agonist
alpha increases BP, beta increases HR
D. is a 40 year old woman who normally has a blood pressure of 110/70 and heart rate of 65 bpm. She has a headache and takes a drug from her medicine cupboard which she thinks is aspirin, but she mistakenly takes another drug from the cupboard. She rests but within an hour she notices that her heart rate has increased. She takes her blood pressure and it has increased to 122/78.
What do you think happened to this patient’s stroke volume?
a. increased
b. decreased
c. no change
a. increased
the beta agonist would increase contractility
D. is a 40 year old woman who normally has a blood pressure of 110/70 and heart rate of 65 bpm. She has a headache and takes a drug from her medicine cupboard which she thinks is aspirin, but she mistakenly takes another drug from the cupboard. She rests but within an hour she notices that her heart rate has increased. She takes her blood pressure and it has increased to 122/78.
If the heart rate had decreased and the blood pressure increased, then which drug would have been more likely?
a. A beta agonist
b. An alpha agonist
c. An alpha antagonist
d. A mixed alpha and beta agonist
e. An angiotensin antagonist
b. An alpha agonist
M.Q. is a 75 year old woman with a history of mitral valve prolapse with mild intermittent regurgitation, but her mitral valve weakens and she suffers from severe acute, mitral regurgitation (assume cardiac output is decreased).
of the following would happen directly in response to the decreased cardiac output?
a. increased heart rate
b. decreased arteriolar constriction
c. decreased arterial pressure
d. decreased ventricular compliance
e. Two of the above
c. decreased arterial pressure
decreased CO directly decreases Pa
M.Q. is a 75 year old woman with a history of mitral valve prolapse with mild intermittent regurgitation, but her mitral valve weakens and she suffers from severe acute, mitral regurgitation (assume cardiac output is decreased).
a result of the change in cardiac output, which of the following would you expect?
a. increased firing in the nerve from the aortic baroreceptor
b. decreased firing in the nerve from the aortic baroreceptor
c. decreased firing in the nerve from the atrial receptors
d. none of the above
b. decreased firing in the nerve from the aortic baroreceptor
decreased Pa decreases firing
M.Q. is a 75 year old woman with a history of mitral valve prolapse with mild intermittent regurgitation, but her mitral valve weakens and she suffers from severe acute, mitral regurgitation (assume cardiac output is decreased).
Which of the following would occur as a reflex response to this event?
a. increased heart rate
b. increased arteriolar constriction
c. increased venous compliance
d. Two of the above
e. none of the above
d. Two of the above
a. increased heart rate (↓ BR firing stimulates increased HR via ↑SNS, ↓PNS)
b. increased arteriolar constriction (↓ BR firing stimulates increased SVR via ↑SNS)
M.Q. is a 75 year old woman with a history of mitral valve prolapse with mild intermittent regurgitation, but her mitral valve weakens and she suffers from severe acute, mitral regurgitation (assume cardiac output is decreased).
Which of the following would cause the reflex response?
a. increased sympathetic nerve activity
b. decreased sympathetic nerve activity
c. increased parasympathetic nerve activity
d. decreased parasympathetic nerve activity
e. Two of the above
e. Two of the above
a. increased sympathetic nerve activity (↓ Pa and ↓BR firing stimulates ↑SNS)
d. decreased parasympathetic nerve activity (↓ Pa and ↓BR firing stimulates ↓ PNS)
TX is a 20 year old man who lives in Gainesville and is visiting Copper Mountain, Colorado (altitude 9700 feet). Soon after driving to town he decides to take a walk on the mountain to see the view. When he reaches the mountaintop he notices that he is breathing more rapidly.
Which of these cardiovascular responses would you expect to occur in Mr X?
a. increased blood pressure and heart rate
b. decreased blood pressure and heart rate
c. increased blood pressure and decreased heart rate
d. decreased blood pressure and increased heart rate
e. no changes would occur in the cardiovascular system because the adaptations are only respiratory
a. increased blood pressure and heart rate (hypoxia leads to ↑SNS)
TX is a 20 year old man who lives in Gainesville and is visiting Copper Mountain, Colorado (altitude 9700 feet). Soon after driving to town he decides to take a walk on the mountain to see the view. When he reaches the mountaintop he notices that he is breathing more rapidly.
Which of the following might also occur in Mr X:
a. arteriolar resistance increases
b. arteriolar resistance decreases
c. venous compliance increases
d. venous compliance decreases
e. Two of the above
e. Two of the above
a. arteriolar resistance increases (↑SNS increases SVR)
d. venous compliance decreases (↑SNS increases venous R (↓C))
JD is a 35 year old man who closes his hand in his car door and faints.
Which of the following is likely to cause the fainting in response to pain:
a. increased sympathetic nerve activity
b. increased vagal parasympathetic nerve activity
c. increased baroreceptor firing
d. increased brain blood flow
e. Two of the above
b. increased vagal parasympathetic nerve activity
↑PNS would slow the heart
JD is a 35 year old man who closes his hand in his car door and faints.
Which of the following should occur in response to the fainting episode?
a. increased sympathetic nerve activity
b. decreased sympathetic nerve activity
c. increased parasympathetic nerve activity
d. Two of the above
a. increased sympathetic nerve activity
(fainting lows Pa and ↓BR firing, causes ↑SNS)
SF is a 29 year old woman who hemorrhages after delivering her new baby and loses 1L of blood.
Which of the following would you expect to occur as a direct effect of the hemorrhage?
a. cardiac output decreases
b. venous compliance increases
c. heart rate increases
d. increased brain blood flow
a. cardiac output decreases
(blood loss decreases VR, leading to dec CO)
SF is a 29 year old woman who hemorrhages after delivering her new baby and loses 1L of blood.
Which of these changes in reflex afferents would you expect to occur in response to the hemorrhage?
a. increased firing in the nerve from the aortic baroreceptor
b. decreased firing in the nerve from the aortic baroreceptor
c. decreased firing in the nerve from the atrial receptors
d increased firing in the nerve from the atrial receptors
e. Two of the above
e. Two of the above
b. decreased firing in the nerve from the aortic baroreceptor (severe blood loss reduces Pa)
c. decreased firing in the nerve from the atrial receptors (blood loss reduces VR)
SF is a 29 year old woman who hemorrhages after delivering her new baby and loses 1L of blood.
Which of the following reflex responses should occur in this patient?
a. decreased heart rate
b. increased arteriolar constriction
c. increased venous compliance
d. Two of the above
e. none of the above
b. increased arteriolar constriction
(↑SNS inc SVR)
Amyl nitrate “poppers” are a drug of abuse and produce flushing and dizziness in the user.
Which of the following is the direct effect of nitrate compounds?
a. decreased heart rate
b. decreased arteriolar constriction
c. increased ventricular contractility
d. Two of the above
e. none of the above
b. decreased arteriolar constriction
NO is a dilator
Amyl nitrate “poppers” are a drug of abuse and produce flushing and dizziness in the user.
Which of the following would occurs as a response to the amyl nitrate effects?
a. increased sympathetic nerve activity
b. decreased sympathetic nerve activity
c. increased parasympathetic nerve activity
d. decreased parasympathetic nerve activity
e. Two of the above
e. Two of the above
a. increased sympathetic nerve activity
d. decreased parasympathetic nerve activity
response to dec Pa is ↑SNS and ↓PNS
Flow in a capillary would be _____________ flow in the aorta.
a. greater than
b. less than
c. the same as
b. less than
In the systemic circulation, normal capillary pressure is ___________ normal venous pressure.
a. greater than
b. less than
c. the same as
a. greater than
Ventricular pressure during systole is ____________ atrial pressure.
a. greater than
b. less than
c. the same as
a. greater than
End diastolic volume is ____________ end systolic volume.
a. greater than
b. less than
c. the same as
a. greater than
In a normal heart, the rate of firing of the SA node cells is __________that of cells in the bundle of His.
a. greater than
b. less than
c. the same as
c. the same as
In the presence of a second degree AV block, the rate of firing of the SA node cells is ______________that of cells in the bundle of His.
a. greater than
b. less than
c. the same as
a. greater than
At rest in a normal person sympathetic tone to the heart is ____________ parasympathetic tone to the heart.
a. greater than
b. less than
c. the same as
b. less than
In ventricular muscle, the contribution of extracellular Ca++ to muscle contraction is
_______________ the contribution of sarcoplasmic reticulum stores of Ca++
a. greater than
b. less than
c. the same as
b. less than
In a patient with coarctation of the thoracic (postductal) aorta, arterial blood pressure in the arm would be _____________ arterial blood pressure in the leg.
a. greater than
b. less than
c. the same as
a. greater than
Which of the following is TRUE about the resistance to blood flow in a vessel in the cardiovascular system?
a. It depends on the compliance of the vessel,
b. It is directly proportional to the radius of the vessel,
c. It is independent of the viscosity of the blood.
d. It is inversely proportional to the fourth power of the radius of the vessel,
e. It can be calculated from the velocity of blood flow in the vessel.
d. It is inversely proportional to the fourth power of the radius of the vessel,
In a normal person, an acute increase in blood pressure will cause heart rate to _____________
a. increase
b. decrease
c. not change
b. decrease
In a normal person, activation of the parasympathetic nervous system will cause heart rate to ____________
a. increase
b. decrease
c. not change
b. decrease
Following arteriolar vasoconstriction, capillary pressure will ____________
a. increase
b. decrease
c. not change
b. decrease
In a normal person, a sudden increase in pericardial fluid which resulted in cardiac tamponade would cause cardiac output to ______________
a. increase
b. decrease
c. not change
b. decrease
In a normal person, a change in arterial blood pressure of 5 mmHg would be expected to cause blood flow to ventricular muscle to ___________
a. increase
b. decrease
c. not change
c. not change
Aortic stenosis would cause ventricular pressure to ____________
a. increase
b. decrease
c. not change
a. increase
Infusion of a drug which is an agonist at a 1 receptors would cause reflex heart rate to __________
a. increase
b. decrease
c. not change
b. decrease
Viagra increases the levels of cGMP in vascular smooth muscle; this would cause the blood flow in the vessels affected by Viagra to ___________
a. increase
b. decrease
c. not change
a. increase
During hemorrhage plasma Angiotensin II levels would ___________
a. increase
b. decrease
c. not change
a. increase
Local release of thromboxane by platelets would cause resistance in the arteriole to _________
a. increase
b. decrease
c. not change
a. increase